The convergence of this series: $sumlimits_{n=2}^infty {1over n^{log n}}$what are some of the methods to do...

Create chunks from an array

Is there a math expression equivalent to the conditional ternary operator?

School performs periodic password audits. Is my password compromised?

Too soon for a plot twist?

What should I do when a paper is published similar to my PhD thesis without citation?

What is Tony Stark injecting into himself in Iron Man 3?

Is there a way to make cleveref distinguish two environments with the same counter?

What do you call someone who likes to pick fights?

I am the person who abides by rules, but breaks the rules. Who am I?

Giving a career talk in my old university, how prominently should I tell students my salary?

Did Amazon pay $0 in taxes last year?

Called into a meeting and told we are being made redundant (laid off) and "not to share outside". Can I tell my partner?

Trocar background-image com delay via jQuery

I reported the illegal activity of my boss to his boss. My boss found out. Now I am being punished. What should I do?

"If + would" conditional in present perfect tense

Help! My Character is too much for her story!

Is it possible to clone a polymorphic object without manually adding overridden clone method into each derived class in C++?

Should we avoid writing fiction about historical events without extensive research?

What will happen if my luggage gets delayed?

Was it really inappropriate to write a pull request for the company I interviewed with?

Movie: boy escapes the real world and goes to a fantasy world with big furry trolls

When an outsider describes family relationships, which point of view are they using?

Use Mercury as quenching liquid for swords?

Why is there an extra space when I type "ls" on the Desktop?



The convergence of this series: $sumlimits_{n=2}^infty {1over n^{log n}}$


what are some of the methods to do divergent/convergent?Determining convergence/divergence with root testConvergence of the series $sum limits_{n=2}^{infty} frac{1}{nlog^s n}$Is $sum_{k=4}^{infty }{k^{log(k)}}/{(log(k))^{k}}$ convergent or divergent?How to prove that if $sum|a_j|<infty$, then $sum|a_j|log(|a_j|^{-1})<infty$Convergence of the series $sumlimits_{n=3}^infty (loglog n)^{-loglog n}$Finding interval of convergence for complicated sumseries convergence with comparison test $frac{1}{nlog(n)^p}$Convergence of series: hintStudying the convergence of the series $sum_{n=1}^inftysinfrac1{n}logleft(1+sinfrac1{n}right)$Convergence of the series $sumlimits_{n=1}^{infty}frac1nlogleft(1+frac1nright)$.Convergence of the series $sum_{n=1}^{infty}frac{(-1)^nsqrt[n]n}{log n}$.













2












$begingroup$


I came across a problem on convergence of series and I did not get into any idea about this -- any help or hints ?
$$sum_{n=2}^infty {1over n^{log n}}$$



What about $n^{log(log n)}$ ?










share|cite|improve this question











$endgroup$












  • $begingroup$
    Making a guess as to what you mean, Baby Rudin 3.28-3.29 would seem helpful.
    $endgroup$
    – ForgotALot
    Mar 7 '16 at 5:59






  • 1




    $begingroup$
    To show divergence if $ale 1$, it is enough (why?) to show divergence when $n=1$. For this use the Integral Test.
    $endgroup$
    – André Nicolas
    Mar 7 '16 at 6:11












  • $begingroup$
    Hello can you make it clear that my question is about n power logn
    $endgroup$
    – Mathslover shah
    Mar 7 '16 at 9:20






  • 3




    $begingroup$
    For some $N$, $n>Nimplieslog(n)>1$. For some other $N$, $n>Nimplieslog(log(n))>1$.
    $endgroup$
    – Yves Daoust
    Mar 7 '16 at 9:24












  • $begingroup$
    Can you edit the problem with n power to the logn please i am not able to do so
    $endgroup$
    – Mathslover shah
    Mar 7 '16 at 9:25
















2












$begingroup$


I came across a problem on convergence of series and I did not get into any idea about this -- any help or hints ?
$$sum_{n=2}^infty {1over n^{log n}}$$



What about $n^{log(log n)}$ ?










share|cite|improve this question











$endgroup$












  • $begingroup$
    Making a guess as to what you mean, Baby Rudin 3.28-3.29 would seem helpful.
    $endgroup$
    – ForgotALot
    Mar 7 '16 at 5:59






  • 1




    $begingroup$
    To show divergence if $ale 1$, it is enough (why?) to show divergence when $n=1$. For this use the Integral Test.
    $endgroup$
    – André Nicolas
    Mar 7 '16 at 6:11












  • $begingroup$
    Hello can you make it clear that my question is about n power logn
    $endgroup$
    – Mathslover shah
    Mar 7 '16 at 9:20






  • 3




    $begingroup$
    For some $N$, $n>Nimplieslog(n)>1$. For some other $N$, $n>Nimplieslog(log(n))>1$.
    $endgroup$
    – Yves Daoust
    Mar 7 '16 at 9:24












  • $begingroup$
    Can you edit the problem with n power to the logn please i am not able to do so
    $endgroup$
    – Mathslover shah
    Mar 7 '16 at 9:25














2












2








2





$begingroup$


I came across a problem on convergence of series and I did not get into any idea about this -- any help or hints ?
$$sum_{n=2}^infty {1over n^{log n}}$$



What about $n^{log(log n)}$ ?










share|cite|improve this question











$endgroup$




I came across a problem on convergence of series and I did not get into any idea about this -- any help or hints ?
$$sum_{n=2}^infty {1over n^{log n}}$$



What about $n^{log(log n)}$ ?







real-analysis sequences-and-series convergence






share|cite|improve this question















share|cite|improve this question













share|cite|improve this question




share|cite|improve this question








edited yesterday









Martin Sleziak

44.8k10119273




44.8k10119273










asked Mar 7 '16 at 5:56









Mathslover shahMathslover shah

110110




110110












  • $begingroup$
    Making a guess as to what you mean, Baby Rudin 3.28-3.29 would seem helpful.
    $endgroup$
    – ForgotALot
    Mar 7 '16 at 5:59






  • 1




    $begingroup$
    To show divergence if $ale 1$, it is enough (why?) to show divergence when $n=1$. For this use the Integral Test.
    $endgroup$
    – André Nicolas
    Mar 7 '16 at 6:11












  • $begingroup$
    Hello can you make it clear that my question is about n power logn
    $endgroup$
    – Mathslover shah
    Mar 7 '16 at 9:20






  • 3




    $begingroup$
    For some $N$, $n>Nimplieslog(n)>1$. For some other $N$, $n>Nimplieslog(log(n))>1$.
    $endgroup$
    – Yves Daoust
    Mar 7 '16 at 9:24












  • $begingroup$
    Can you edit the problem with n power to the logn please i am not able to do so
    $endgroup$
    – Mathslover shah
    Mar 7 '16 at 9:25


















  • $begingroup$
    Making a guess as to what you mean, Baby Rudin 3.28-3.29 would seem helpful.
    $endgroup$
    – ForgotALot
    Mar 7 '16 at 5:59






  • 1




    $begingroup$
    To show divergence if $ale 1$, it is enough (why?) to show divergence when $n=1$. For this use the Integral Test.
    $endgroup$
    – André Nicolas
    Mar 7 '16 at 6:11












  • $begingroup$
    Hello can you make it clear that my question is about n power logn
    $endgroup$
    – Mathslover shah
    Mar 7 '16 at 9:20






  • 3




    $begingroup$
    For some $N$, $n>Nimplieslog(n)>1$. For some other $N$, $n>Nimplieslog(log(n))>1$.
    $endgroup$
    – Yves Daoust
    Mar 7 '16 at 9:24












  • $begingroup$
    Can you edit the problem with n power to the logn please i am not able to do so
    $endgroup$
    – Mathslover shah
    Mar 7 '16 at 9:25
















$begingroup$
Making a guess as to what you mean, Baby Rudin 3.28-3.29 would seem helpful.
$endgroup$
– ForgotALot
Mar 7 '16 at 5:59




$begingroup$
Making a guess as to what you mean, Baby Rudin 3.28-3.29 would seem helpful.
$endgroup$
– ForgotALot
Mar 7 '16 at 5:59




1




1




$begingroup$
To show divergence if $ale 1$, it is enough (why?) to show divergence when $n=1$. For this use the Integral Test.
$endgroup$
– André Nicolas
Mar 7 '16 at 6:11






$begingroup$
To show divergence if $ale 1$, it is enough (why?) to show divergence when $n=1$. For this use the Integral Test.
$endgroup$
– André Nicolas
Mar 7 '16 at 6:11














$begingroup$
Hello can you make it clear that my question is about n power logn
$endgroup$
– Mathslover shah
Mar 7 '16 at 9:20




$begingroup$
Hello can you make it clear that my question is about n power logn
$endgroup$
– Mathslover shah
Mar 7 '16 at 9:20




3




3




$begingroup$
For some $N$, $n>Nimplieslog(n)>1$. For some other $N$, $n>Nimplieslog(log(n))>1$.
$endgroup$
– Yves Daoust
Mar 7 '16 at 9:24






$begingroup$
For some $N$, $n>Nimplieslog(n)>1$. For some other $N$, $n>Nimplieslog(log(n))>1$.
$endgroup$
– Yves Daoust
Mar 7 '16 at 9:24














$begingroup$
Can you edit the problem with n power to the logn please i am not able to do so
$endgroup$
– Mathslover shah
Mar 7 '16 at 9:25




$begingroup$
Can you edit the problem with n power to the logn please i am not able to do so
$endgroup$
– Mathslover shah
Mar 7 '16 at 9:25










2 Answers
2






active

oldest

votes


















1












$begingroup$

For which $a$ does
$sum_{n=2}^infty {1over n^a}
$
converge?
For which $a$ does it diverge?



If this converges,
then
$sum_{n=2}^infty {1over n^alog n}$
will certainly converge
(do you see why?).






share|cite|improve this answer









$endgroup$





















    1












    $begingroup$

    Since the series is a positive one and the general term's sequence is monotonically decreasing to zero, you can use Cauchy's Condensation Test, and:



    $$frac{2^n}{(2^n)^{log2^n}}=frac{2^n}{2^{n^2log2}}=frac1{2^{nleft(nlog2-1right)}}lefrac1{2^n}$$






    share|cite|improve this answer









    $endgroup$













    • $begingroup$
      Aaaa that is there . Thanks .
      $endgroup$
      – Mathslover shah
      Mar 7 '16 at 10:16










    • $begingroup$
      Thank you to anyone who downvoted/upvoted this. It would really help me if anyone who downvoted could be please kind enough to write down the reason: is something wrong, or unclear? I try to learn of my mistakes...if I recognize them.
      $endgroup$
      – DonAntonio
      Mar 7 '16 at 10:32











    Your Answer





    StackExchange.ifUsing("editor", function () {
    return StackExchange.using("mathjaxEditing", function () {
    StackExchange.MarkdownEditor.creationCallbacks.add(function (editor, postfix) {
    StackExchange.mathjaxEditing.prepareWmdForMathJax(editor, postfix, [["$", "$"], ["\\(","\\)"]]);
    });
    });
    }, "mathjax-editing");

    StackExchange.ready(function() {
    var channelOptions = {
    tags: "".split(" "),
    id: "69"
    };
    initTagRenderer("".split(" "), "".split(" "), channelOptions);

    StackExchange.using("externalEditor", function() {
    // Have to fire editor after snippets, if snippets enabled
    if (StackExchange.settings.snippets.snippetsEnabled) {
    StackExchange.using("snippets", function() {
    createEditor();
    });
    }
    else {
    createEditor();
    }
    });

    function createEditor() {
    StackExchange.prepareEditor({
    heartbeatType: 'answer',
    autoActivateHeartbeat: false,
    convertImagesToLinks: true,
    noModals: true,
    showLowRepImageUploadWarning: true,
    reputationToPostImages: 10,
    bindNavPrevention: true,
    postfix: "",
    imageUploader: {
    brandingHtml: "Powered by u003ca class="icon-imgur-white" href="https://imgur.com/"u003eu003c/au003e",
    contentPolicyHtml: "User contributions licensed under u003ca href="https://creativecommons.org/licenses/by-sa/3.0/"u003ecc by-sa 3.0 with attribution requiredu003c/au003e u003ca href="https://stackoverflow.com/legal/content-policy"u003e(content policy)u003c/au003e",
    allowUrls: true
    },
    noCode: true, onDemand: true,
    discardSelector: ".discard-answer"
    ,immediatelyShowMarkdownHelp:true
    });


    }
    });














    draft saved

    draft discarded


















    StackExchange.ready(
    function () {
    StackExchange.openid.initPostLogin('.new-post-login', 'https%3a%2f%2fmath.stackexchange.com%2fquestions%2f1686630%2fthe-convergence-of-this-series-sum-limits-n-2-infty-1-over-n-log-n%23new-answer', 'question_page');
    }
    );

    Post as a guest















    Required, but never shown

























    2 Answers
    2






    active

    oldest

    votes








    2 Answers
    2






    active

    oldest

    votes









    active

    oldest

    votes






    active

    oldest

    votes









    1












    $begingroup$

    For which $a$ does
    $sum_{n=2}^infty {1over n^a}
    $
    converge?
    For which $a$ does it diverge?



    If this converges,
    then
    $sum_{n=2}^infty {1over n^alog n}$
    will certainly converge
    (do you see why?).






    share|cite|improve this answer









    $endgroup$


















      1












      $begingroup$

      For which $a$ does
      $sum_{n=2}^infty {1over n^a}
      $
      converge?
      For which $a$ does it diverge?



      If this converges,
      then
      $sum_{n=2}^infty {1over n^alog n}$
      will certainly converge
      (do you see why?).






      share|cite|improve this answer









      $endgroup$
















        1












        1








        1





        $begingroup$

        For which $a$ does
        $sum_{n=2}^infty {1over n^a}
        $
        converge?
        For which $a$ does it diverge?



        If this converges,
        then
        $sum_{n=2}^infty {1over n^alog n}$
        will certainly converge
        (do you see why?).






        share|cite|improve this answer









        $endgroup$



        For which $a$ does
        $sum_{n=2}^infty {1over n^a}
        $
        converge?
        For which $a$ does it diverge?



        If this converges,
        then
        $sum_{n=2}^infty {1over n^alog n}$
        will certainly converge
        (do you see why?).







        share|cite|improve this answer












        share|cite|improve this answer



        share|cite|improve this answer










        answered Mar 7 '16 at 6:10









        marty cohenmarty cohen

        74.2k549128




        74.2k549128























            1












            $begingroup$

            Since the series is a positive one and the general term's sequence is monotonically decreasing to zero, you can use Cauchy's Condensation Test, and:



            $$frac{2^n}{(2^n)^{log2^n}}=frac{2^n}{2^{n^2log2}}=frac1{2^{nleft(nlog2-1right)}}lefrac1{2^n}$$






            share|cite|improve this answer









            $endgroup$













            • $begingroup$
              Aaaa that is there . Thanks .
              $endgroup$
              – Mathslover shah
              Mar 7 '16 at 10:16










            • $begingroup$
              Thank you to anyone who downvoted/upvoted this. It would really help me if anyone who downvoted could be please kind enough to write down the reason: is something wrong, or unclear? I try to learn of my mistakes...if I recognize them.
              $endgroup$
              – DonAntonio
              Mar 7 '16 at 10:32
















            1












            $begingroup$

            Since the series is a positive one and the general term's sequence is monotonically decreasing to zero, you can use Cauchy's Condensation Test, and:



            $$frac{2^n}{(2^n)^{log2^n}}=frac{2^n}{2^{n^2log2}}=frac1{2^{nleft(nlog2-1right)}}lefrac1{2^n}$$






            share|cite|improve this answer









            $endgroup$













            • $begingroup$
              Aaaa that is there . Thanks .
              $endgroup$
              – Mathslover shah
              Mar 7 '16 at 10:16










            • $begingroup$
              Thank you to anyone who downvoted/upvoted this. It would really help me if anyone who downvoted could be please kind enough to write down the reason: is something wrong, or unclear? I try to learn of my mistakes...if I recognize them.
              $endgroup$
              – DonAntonio
              Mar 7 '16 at 10:32














            1












            1








            1





            $begingroup$

            Since the series is a positive one and the general term's sequence is monotonically decreasing to zero, you can use Cauchy's Condensation Test, and:



            $$frac{2^n}{(2^n)^{log2^n}}=frac{2^n}{2^{n^2log2}}=frac1{2^{nleft(nlog2-1right)}}lefrac1{2^n}$$






            share|cite|improve this answer









            $endgroup$



            Since the series is a positive one and the general term's sequence is monotonically decreasing to zero, you can use Cauchy's Condensation Test, and:



            $$frac{2^n}{(2^n)^{log2^n}}=frac{2^n}{2^{n^2log2}}=frac1{2^{nleft(nlog2-1right)}}lefrac1{2^n}$$







            share|cite|improve this answer












            share|cite|improve this answer



            share|cite|improve this answer










            answered Mar 7 '16 at 9:58









            DonAntonioDonAntonio

            179k1494233




            179k1494233












            • $begingroup$
              Aaaa that is there . Thanks .
              $endgroup$
              – Mathslover shah
              Mar 7 '16 at 10:16










            • $begingroup$
              Thank you to anyone who downvoted/upvoted this. It would really help me if anyone who downvoted could be please kind enough to write down the reason: is something wrong, or unclear? I try to learn of my mistakes...if I recognize them.
              $endgroup$
              – DonAntonio
              Mar 7 '16 at 10:32


















            • $begingroup$
              Aaaa that is there . Thanks .
              $endgroup$
              – Mathslover shah
              Mar 7 '16 at 10:16










            • $begingroup$
              Thank you to anyone who downvoted/upvoted this. It would really help me if anyone who downvoted could be please kind enough to write down the reason: is something wrong, or unclear? I try to learn of my mistakes...if I recognize them.
              $endgroup$
              – DonAntonio
              Mar 7 '16 at 10:32
















            $begingroup$
            Aaaa that is there . Thanks .
            $endgroup$
            – Mathslover shah
            Mar 7 '16 at 10:16




            $begingroup$
            Aaaa that is there . Thanks .
            $endgroup$
            – Mathslover shah
            Mar 7 '16 at 10:16












            $begingroup$
            Thank you to anyone who downvoted/upvoted this. It would really help me if anyone who downvoted could be please kind enough to write down the reason: is something wrong, or unclear? I try to learn of my mistakes...if I recognize them.
            $endgroup$
            – DonAntonio
            Mar 7 '16 at 10:32




            $begingroup$
            Thank you to anyone who downvoted/upvoted this. It would really help me if anyone who downvoted could be please kind enough to write down the reason: is something wrong, or unclear? I try to learn of my mistakes...if I recognize them.
            $endgroup$
            – DonAntonio
            Mar 7 '16 at 10:32


















            draft saved

            draft discarded




















































            Thanks for contributing an answer to Mathematics Stack Exchange!


            • Please be sure to answer the question. Provide details and share your research!

            But avoid



            • Asking for help, clarification, or responding to other answers.

            • Making statements based on opinion; back them up with references or personal experience.


            Use MathJax to format equations. MathJax reference.


            To learn more, see our tips on writing great answers.




            draft saved


            draft discarded














            StackExchange.ready(
            function () {
            StackExchange.openid.initPostLogin('.new-post-login', 'https%3a%2f%2fmath.stackexchange.com%2fquestions%2f1686630%2fthe-convergence-of-this-series-sum-limits-n-2-infty-1-over-n-log-n%23new-answer', 'question_page');
            }
            );

            Post as a guest















            Required, but never shown





















































            Required, but never shown














            Required, but never shown












            Required, but never shown







            Required, but never shown

































            Required, but never shown














            Required, but never shown












            Required, but never shown







            Required, but never shown







            Popular posts from this blog

            Magento 2 - Add success message with knockout Planned maintenance scheduled April 23, 2019 at 23:30 UTC (7:30pm US/Eastern) Announcing the arrival of Valued Associate #679: Cesar Manara Unicorn Meta Zoo #1: Why another podcast?Success / Error message on ajax request$.widget is not a function when loading a homepage after add custom jQuery on custom themeHow can bind jQuery to current document in Magento 2 When template load by ajaxRedirect page using plugin in Magento 2Magento 2 - Update quantity and totals of cart page without page reload?Magento 2: Quote data not loaded on knockout checkoutMagento 2 : I need to change add to cart success message after adding product into cart through pluginMagento 2.2.5 How to add additional products to cart from new checkout step?Magento 2 Add error/success message with knockoutCan't validate Post Code on checkout page

            Fil:Tokke komm.svg

            Where did Arya get these scars? Unicorn Meta Zoo #1: Why another podcast? Announcing the arrival of Valued Associate #679: Cesar Manara Favourite questions and answers from the 1st quarter of 2019Why did Arya refuse to end it?Has the pronunciation of Arya Stark's name changed?Has Arya forgiven people?Why did Arya Stark lose her vision?Why can Arya still use the faces?Has the Narrow Sea become narrower?Does Arya Stark know how to make poisons outside of the House of Black and White?Why did Nymeria leave Arya?Why did Arya not kill the Lannister soldiers she encountered in the Riverlands?What is the current canonical age of Sansa, Bran and Arya Stark?